K
Khách

Hãy nhập câu hỏi của bạn vào đây, nếu là tài khoản VIP, bạn sẽ được ưu tiên trả lời.

NV
18 tháng 11 2019

Chỉ có biến đổi tương đương:

\(\frac{x^2+y^2+2}{\left(1+x^2\right)\left(1+y^2\right)}\le\frac{2}{1+xy}\Leftrightarrow\left(1+xy\right)\left(x^2+y^2+2\right)\le2\left(1+x^2\right)\left(1+y^2\right)\)

\(\Leftrightarrow x^2+y^2+2+x^3y+xy^3+2xy\le2+2x^2+2y^2+2x^2y^2\)

\(\Leftrightarrow xy\left(x^2+y^2-2xy\right)-\left(x^2-2xy+y^2\right)\le0\)

\(\Leftrightarrow\left(xy-1\right)\left(x-y\right)^2\le0\) (luôn đúng với mọi \(xy\le1\))

Dấu "=" xảy ra khi \(\left[{}\begin{matrix}x=y\\xy=1\end{matrix}\right.\)

b/ Tính chất của z ở câu b là gì bạn? z bất kì là ko được đâu, hơn nữa mẫu số của vế phải thấy hơi kì quặc

18 tháng 11 2019

Phần b là mình đánh nhầm sửa lại là \(\frac{1}{1+xyz}\)

9 tháng 3 2020

Áp dụng BĐT Cosi cho 2 sô dương ta có: \(x^2+yz\ge2x\sqrt{yz}\)

Tương tự: \(y^2+zx\ge2y\sqrt{zx};z^2+xy\ge2z\sqrt{xy}\)

Khi đó BĐT sẽ được chứng minh nếu ta chỉ ra được:

\(\frac{1}{2x\sqrt{yz}}+\frac{1}{2y\sqrt{zx}}+\frac{1}{2z\sqrt{xy}}\le\frac{1}{2}\left(\frac{1}{xy}+\frac{1}{yz}+\frac{1}{zx}\right)\)

\(\Leftrightarrow\frac{\sqrt{xy}+\sqrt{yz}+\sqrt{zx}}{xyz}\le\frac{x+y+z}{xyz}\Leftrightarrow\sqrt{xy}+\sqrt{yz}+\sqrt{zx}\le x+y+z\)

\(\Leftrightarrow\frac{1}{2}\left(\left(\sqrt{x}-\sqrt{y}\right)^2+\left(\sqrt{y}-\sqrt{z}\right)^2+\left(\sqrt{z}-\sqrt{x}\right)^2\right)\ge0\)(luôn đúng)

Dấu "=" xảy ra khi \(x=y=z\)

9 tháng 3 2020

Áp dụng BĐT Cosi cho 2 số dương ta có: \(x^2+yz\ge2\sqrt{x^2yz}=2x\sqrt{yz}\)

Tương tự: \(y^2+zx\ge2y\sqrt{zx},z^2+xy\ge2z\sqrt{xy}\)

Khi đó BĐT sẽ được chứng minh nếu ta chỉ ra được: 

\(\frac{1}{2x\sqrt{yz}}+\frac{1}{2y\sqrt{zx}}+\frac{1}{2z\sqrt{xy}}\le\frac{1}{2}\left(\frac{1}{xy}+\frac{1}{yz}+\frac{1}{zx}\right)\)

\(\Leftrightarrow\frac{\sqrt{xy}+\sqrt{yz}+\sqrt{zx}}{xyz}\le\frac{x+y+z}{xyz}\Leftrightarrow\sqrt{xy}+\sqrt{yz}+\sqrt{zx}\le x+y+z\)

\(\Leftrightarrow\frac{1}{2}\left(\left(\sqrt{x}-\sqrt{y}\right)^2+\left(\sqrt{y}-\sqrt{z}\right)^2+\left(\sqrt{z}-\sqrt{x}\right)^2\right)\ge0\)(luôn đúng)

Vậy BĐT được chứng minh. Dấu "=" xảy ra khi \(x=y=z\)

Cách 2:

Ta chuẩn hóa xyz=1

BĐT viết lại là \(\frac{x}{x^3+1}+\frac{y}{y^3+1}+\frac{z}{z^3+1}\le\frac{1}{2}\left(x+y+z\right)\)

Ta sử dụng đánh giá

\(x-\frac{2x}{x^3+1}+\frac{3}{2}\ge\frac{9x^2}{2\left(x^2+x+1\right)}\)\(\Leftrightarrow\frac{\left(x-1\right)^2\left(2x^4+3x^2+7x+3\right)}{2\left(x^3+1\right)\left(x^2+x+1\right)}\ge0\)

Do vậy ta cần c/m \(\frac{x^2}{x^2+x+1}+\frac{y^2}{y^2+y+1}+\frac{z^2}{z^2+z+1}\ge1\)

 ta có \(\left(x;y;z\right)\rightarrow\left(\frac{a^2}{bc};\frac{b^2}{ca};\frac{c^2}{ab}\right)\)

BĐT viết lại là \(\frac{a^4}{a^4+a^2bc+\left(bc\right)^2}+\frac{b^4}{b^4+b^2ca+\left(ca\right)^2}+\frac{c^4}{c^4+c^2ab+\left(ab\right)^2}\ge1\)

Theo bđt Cauchy-Schwarz ta có

\(VT\ge\frac{\left(a^2+b^2+c^2\right)^2}{a^4+b^4+c^4+abc\left(a+b+c\right)+\left(ab\right)^2+\left(bc\right)^2+\left(ca\right)^2}\)

Theo bđt AM-GM ta có

\(VT\ge\frac{\left(a^2+b^2+c^2\right)^2}{a^4+b^4+c^4+2\left(ab\right)^2+2\left(bc\right)^2+2\left(ca\right)^2}=1\)

Dấu "=" xảy ra khi a=b=c=> x=y=z

15 tháng 6 2017

a/ \(\frac{1}{1+x}+\frac{1}{1+y}\le\frac{2}{1+\sqrt{xy}}\)

\(\Leftrightarrow\left(1+x\right)\left(1+\sqrt{xy}\right)+\left(1+y\right)\left(1+\sqrt{xy}\right)-2\left(1+x\right)\left(1+y\right)\le0\)

\(\Leftrightarrow x\sqrt{xy}+2\sqrt{xy}+y\sqrt{xy}-x-y-2xy\le0\)

\(\Leftrightarrow\sqrt{xy}\left(x-2\sqrt{xy}+y\right)-\left(x-2\sqrt{xy}+y\right)\le0\)

\(\Leftrightarrow\left(\sqrt{x}-\sqrt{y}\right)^2\left(\sqrt{xy}-1\right)\le0\) đúng vì \(x,y\le1\)

b/ Vì \(\hept{\begin{cases}0\le x\le y\le z\le t\\yt\le1\end{cases}}\)

\(\Rightarrow\hept{\begin{cases}xz\le1\\yt\le1\end{cases}}\)

Áp dụng câu a ta được

\(\frac{1}{1+x}+\frac{1}{1+y}+\frac{1}{1+z}+\frac{1}{1+t}\le\frac{2}{1+\sqrt{xz}}+\frac{2}{1+\sqrt{yt}}\le\frac{4}{1+\sqrt[4]{xyzt}}\)

15 tháng 6 2017

khó quá

AH
Akai Haruma
Giáo viên
27 tháng 6 2020

Lời giải:

Do $x,y,z\in [0;1]$ nên $1+yz; 1+xz; 1+xy\geq 1+xyz$

$\Rightarrow \frac{x}{1+yz}+\frac{y}{1+xz}+\frac{z}{1+xy}\leq \frac{x+y+z}{1+xyz}$

Ta cần chứng minh: $\frac{x+y+z}{1+xyz}\leq 2$

$\Leftrightarrow x+y+z\leq 2+2xyz(*)$

Thật vậy:

$x,y\in [0;1]\Rightarrow (x-1)(y-1)\geq 0$

$\Leftrightarrow xy+1\geq x+y\Rightarrow xy+z+1\geq x+y+z(1)$
Mà:

$xy+z+1-(2+2xyz)=xy+z-2xyz-1=xy(1-z)-(1-z)-xyz=(xy-1)(1-z)-xyz\leq 0$ do $0\leq x,y,z\leq 1$)

$\Rightarrow xy+z+1\leq 2+2xyz(2)$

Từ $(1);(2)\Rightarrow x+y+z\leq 2+2xyz$

BĐT $(*)$ đc chứng minh nên ta có đpcm.

Dấu "=" xảy ra khi $(x,y,z)=(1,1,0)$ và hoán vị

1 tháng 7 2020

Trâu bò nhưng bù lại là đơn giản:

\(VP-VT\equiv f\left(x,y,z\right)=f\left(\frac{a}{a+1},\frac{b}{b+1},\frac{c}{c+1}\right)\ge0\)

Bất đẳng thức cuối quy đồng lên sẽ thấy điều hiển nhiên ;)

7 tháng 3 2018

Áp dụng BĐT AM-GM ta có: 

\(VT=\sqrt{\frac{xy}{z+xy}}+\sqrt{\frac{xz}{y+xz}}+\sqrt{\frac{yz}{x+yz}}\)

\(=\sqrt{\frac{xy}{z\left(x+y+z\right)+xy}}+\sqrt{\frac{xz}{y\left(x+y+z\right)+xz}}+\sqrt{\frac{yz}{x\left(x+y+z\right)+yz}}\)

\(=\sqrt{\frac{xy}{\left(x+z\right)\left(y+z\right)}}+\sqrt{\frac{xz}{\left(x+y\right)\left(y+z\right)}}+\sqrt{\frac{yz}{\left(x+y\right)\left(x+z\right)}}\)

\(\le\frac{1}{2}\left(\frac{x}{x+z}+\frac{y}{y+z}+\frac{x}{x+y}+\frac{z}{y+z}+\frac{y}{x+y}+\frac{z}{x+z}\right)\)

\(=\frac{1}{2}\left(\frac{x+z}{x+z}+\frac{y+z}{y+z}+\frac{x+y}{x+y}\right)=\frac{3}{2}\)

Dấu "=" <=> \(x=y=z=\frac{1}{3}\)

Ủng hộ và kb với mình ha ^^

6 tháng 3 2018
Từ gt suy ra z=1-x-y Thầy vào sau đó áp dụng AM-GM
28 tháng 11 2017

Với: \(x;y\le1\)

Chứng minh rằng: \(\frac{1}{1+x^2}+\frac{1}{1+y^2}\le\frac{2}{1+xy}\)

\(\Leftrightarrow\left(\frac{1}{1+x^2}-\frac{1}{1+xy}\right)+\left(\frac{1}{1+y^2}-\frac{1}{1+xy}\right)\le0\)

\(\Leftrightarrow\frac{1+xy-\left(1+x^2\right)}{\left(1+x^2\right)\left(1+xy\right)}+\frac{1+xy-\left(1+y^2\right)}{\left(1+y^2\right)\left(1+xy\right)}\le0\)

\(\Leftrightarrow\frac{-x\left(x-y\right)-\left(1+x^2\right)}{\left(1+x^2\right)\left(1+xy\right)+\left(1+y^2\right)}+\frac{y\left(x-y\right)\left(1+x^2\right)}{\left(1+y^2\right)\left(1+xy\right)\left(1+x^2\right)}\le0\)

\(\Leftrightarrow\left(x-y\right)\left(-x+y-xy^2+x^2y\right)\le0\)

\(\Leftrightarrow\left(x-y\right)^2\left(xy-1\right)\ge\)(Luôn đúng \(\forall x,y\le1\))

\(\RightarrowĐPCM\)

P/s: Sai đâu thì sửa nhé!

28 tháng 11 2017

Với: x;y ≤ 1 Chứng minh rằng:

1 + x 2 1 + 1 + y 2 1 ≤ 1 + xy 2 ⇔ 1 + x 2 1 − 1 + xy 1 + 1 + y 2 1 − 1 + xy 1 ≤ 0

⇔ 1 + x 2 1 + xy 1 + xy − 1 + x 2 + 1 + y 2 1 + xy 1 + xy − 1 + y 2 ≤ 0

⇔ 1 + x 2 1 + xy + 1 + y 2 −x x − y − 1 + x 2 + 1 + y 2 1 + xy 1 + x 2 y x − y 1 + x 2 ≤ 0

⇔ x − y −x + y − xy 2 + x 2 y ≤ 0

24 tháng 1 2018

v~~ ko thằng admin :(( t làm cái bài này mất gần 30 phút mà bây giờ nó éo hiện câu trả lời của tao ???? hận quá đi 

24 tháng 1 2018

bài này easy lắm bạn ơi :(( 

áp dụng BDT (Am-ag) mẫu ta có

\(\left(x^2+y^2\right)\ge2\sqrt{x^2y^2}=2xy\) rồi thay vào

suy ra   \(\frac{1}{x^2+y^2+2}\le\frac{1}{2xy+2}\)

\(\left(y^2+z^2\right)\ge2yz\)

suy ra \(\frac{1}{y^2+z^2+2}\le\frac{1}{2yz+2}\)

tượng tự vs  BDT con lại rồi + vế vs vế ta được

\(VT\le\frac{1}{2xy+2}+\frac{1}{2yz+2}+\frac{1}{2xz+2}=\frac{1}{xy+xy+1+1}+\frac{1}{yz+yz+1+1}+\frac{1}{xz+xz+1+1}\)

gọi cái  \(\frac{1}{yz+yz+1+1}+.........=Pain\)

áp dụng cosi sáp cho 4 số ta được

\(\frac{1}{xy+xy+1+1}\le\frac{1}{16}\left(\frac{1}{xy}+\frac{1}{xy}+\frac{1}{1}+\frac{1}{1}\right)\)

\(\frac{1}{yz+yz+1+1}\le\frac{1}{16}\left(\frac{1}{yz}+\frac{1}{yz}+\frac{1}{1}+\frac{1}{1}\right)\)

\(\frac{1}{xz+xz+1+1}\le\frac{1}{16}\left(\frac{1}{xz}+\frac{1}{xz}+\frac{1}{1}+\frac{1}{1}\right)\)

+ vế với vế ta được

\(VT\le Pain\le\frac{1}{16}\left(\frac{2}{xz}+\frac{2}{yz}+\frac{2}{xy}+\frac{2}{2}+\frac{2}{2}+\frac{2}{2}\right)\)

\(VT\le PAIN\le\frac{1}{8}\left(\frac{1}{xz}+\frac{1}{yz}+\frac{1}{xy}+1+1+1\right)\)

bây giờ m đi chứng minh cái \(\frac{1}{zy}+\frac{1}{yz}+\frac{1}{xy}\ge3\) chắc là m làm được

áp dụng BDT cô si ta có

\(\frac{1}{xz}+xz\ge2\)

\(\frac{1}{yz}+yz\ge2\)

\(\frac{1}{xz}+zx\ge2\)

+ vế với vế ta được

\(\frac{1}{xy}+\frac{1}{yz}+\frac{1}{zx}+xy+yz+zx\ge6\)

mà đề bài cho xy+yz+xz=3 suy ra

\(\frac{1}{xy}+\frac{1}{yz}+\frac{1}{zx}\ge3\)

nhưng mà nó trái dấu oy :(( kệ nhé cứ thay vào nhé không sao hết bạn oy :)

thay vào ta được

\(VT\le PAIN\le\frac{1}{8}\left(3+3\right)=\frac{3}{4}\)

ĐIỀU CẦN PHẢI CHỨNG MINH :(( 

AH
Akai Haruma
Giáo viên
25 tháng 1 2018

Lời giải:

Ta có:

\(\text{VT}=\frac{1}{x^2+y^2+2}+\frac{1}{y^2+z^2+2}+\frac{1}{z^2+x^2+2}\)

\(\Rightarrow 2\text{VT}=\frac{2}{x^2+y^2+2}+\frac{2}{y^2+z^2+2}+\frac{2}{z^2+x^2+2}\)

\(2\text{VT}=1-\frac{x^2+y^2}{x^2+y^2+2}+1-\frac{y^2+z^2}{y^2+z^2+2}+1-\frac{z^2+x^2}{z^2+x^2+2}\)

\(2\text{VT}=3-\left(\frac{x^2+y^2}{x^2+y^2+2}+\frac{y^2+z^2}{y^2+z^2+2}+\frac{z^2+x^2}{z^2+x^2+2}\right)=3-A\)

Áp dụng BĐT Cauchy-Schwarz:

\(A\geq \frac{(\sqrt{x^2+y^2}+\sqrt{y^2+z^2}+\sqrt{z^2+x^2})^2}{2(x^2+y^2+z^2)+6}=\frac{(\sqrt{x^2+y^2}+\sqrt{y^2+z^2}+\sqrt{z^2+x^2})^2}{2(x^2+y^2+z^2+xy+yz+xz)}(*)\)

Xét tử số:

\((\sqrt{x^2+y^2}+\sqrt{y^2+z^2}+\sqrt{z^2+x^2})^2\)

\(=2(x^2+y^2+z^2)+2(\sqrt{(x^2+y^2)(x^2+z^2)}+\sqrt{(x^2+y^2)(y^2+z^2)}+\sqrt{(y^2+z^2)(z^2+x^2)})\)

Áp dụng BĐT Bunhiacopxky:

\(\sqrt{(x^2+y^2)(x^2+z^2)}\geq \sqrt{(x^2+yz)^2}=x^2+yz\)

\(\sqrt{(x^2+y^2)(y^2+z^2)}\geq \sqrt{(xz+y^2)^2}=xz+y^2\)

\(\sqrt{(y^2+z^2)(z^2+x^2)}\geq \sqrt{(z^2+xy)^2}=z^2+xy\)

\(\Rightarrow \sum \sqrt{(x^2+y^2)(x^2+z^2)}\geq x^2+y^2+z^2+xy+yz+xz\)

\(\Rightarrow (\sqrt{x^2+y^2}+\sqrt{y^2+z^2}+\sqrt{z^2+x^2})^2\geq 4(x^2+y^2+z^2)+2(xy+yz+xz)\)

\(\geq 3(x^2+y^2+z^2)+3(xy+yz+xz)=3(x^2+y^2+z^2+xy+yz+xz)\)

(theo BĐT AM-GM)

Do đó: Từ \((*)\Rightarrow A\geq \frac{3(x^2+y^2+z^2+xy+yz+xz)}{2(x^2+y^2+z^2+xy+yz+xz)}=\frac{3}{2}\)

\(\Rightarrow 2\text{VT}\leq 3-\frac{3}{2}=\frac{3}{2}\)

\(\Rightarrow \text{VT}\leq \frac{3}{4}\) (đpcm)

Dấu bằng xảy ra khi \(x=y=z=1\)

26 tháng 1 2018

We have: \(\dfrac{1}{x^2+y^2+2}=\dfrac{1}{x^2+y^2+z^2+2-z^2}\le\dfrac{1}{5-z^2}\)

Similarly and by adding them:

\(\dfrac{1}{5-x^2}+\dfrac{1}{5-y^2}+\dfrac{1}{5-z^2}\le\dfrac{3}{4}\left(\circledast\right)\)

We know that \(\dfrac{1}{5-x^2}\le\dfrac{3\left(x^2+x\right)}{8\left(x^2+x+1\right)}\)

\(\Leftrightarrow-\dfrac{\left(x-1\right)^2\left(3x^2+9x+8\right)}{8\left(x^2-5\right)\left(x^2+x+1\right)}\le0\) It's obviously

\(\Rightarrow L.H.S_{\left(\circledast\right)}\le\dfrac{3}{8}\left(\dfrac{x^2+x}{x^2+x+1}+\dfrac{y^2+y}{y^2+y+1}+\dfrac{z^2+z}{z^2+z+1}\right)\le\dfrac{3}{4}\)

The equality occur when \(x=y=z=1\)

Done!